LSAT and Law School Admissions Forum

Get expert LSAT preparation and law school admissions advice from PowerScore Test Preparation.

 Jon Denning
PowerScore Staff
  • PowerScore Staff
  • Posts: 904
  • Joined: Apr 11, 2011
|
#40572
Complete Question Explanation
(The complete setup for this game can be found here: lsat/viewtopic.php?t=14969)

The correct answer is (C)

If F and I are in a group, then that group also includes L (from the second rule where F and L are always together). As we then think about our distribution options, the 4-1-1-1 possibility is out: I must be a single in that distribution, with the 4-part group consisting of F, L, M, and H.

Instead, we must be in a 3-2-1-1 distribution, with the FLI group as the triple, K as a single, and the other three candidates—G, H, and M—split into a double and single, remembering of course that G always goes to U.

So we could potentially put H or M with G on U, or we could leave G single and put H and M together elsewhere.

In short, we still have quite a few options placement-wise (no surprise really, considering that each of the four wrong answers in this question could be true...i.e. there's a lot of possibility allowed for by the answers themselves).

So what do we know for sure then?

Well, the FLI block has to go to either R or S. Putting that 3-person block on T would violate the S > T rule (the last rule), and putting the 3-person block on U would group it with G and give us a 4-1-1-1 distribution, where S and T are both singles (also breaking the last rule).

And voila! The right answer, answer choice (C), describes that impossibility: putting L on U means the whole FLI block goes there with G, and that can't happen for the reason described in the last paragraph!

The other four answers are all allowed:

Answer choice (A): As I mention above, H and G can go together. That would leave M and K as singles (one of which is on T and the other on R). FLI would go with S.

Answer choice (B): K, our always-alone variable, can go to T without issue. The FLI block could then go to either R or S and be fine.

Answer choice (D): If M goes to S, then H has to go with it so that S can have 2 people and be greater than T. That means our singles are K on T, and G on U, and the three-person block FLI goes to R.

Answer choice (E): Like answer choice (A), M and G can go together. That simply makes H and K our singles, one of which, again, is assigned to T and the other assigned to R. FLI would be assigned to S in this scenario.
 medialaw111516
  • Posts: 80
  • Joined: Dec 11, 2018
|
#71903
So for this one, I looked at my diagram for question 9 where F and I were both evaluated by S, and I immediately zeroed in on C as the correct answer because you can't separate L and F. Is that a good way to go about this question, or would it not work because the last question had a location condition of M being alone?
 Claire Horan
PowerScore Staff
  • PowerScore Staff
  • Posts: 408
  • Joined: Apr 18, 2016
|
#71921
Hi MediaLaw,

It sounds like you've answered your own question! In general, you should not use local diagrams to answer questions where the local condition is present. However, it sounds like what happened here was that Question 9 reminded you of one of the rules, that F and L are a block.

A more straightforward way to solve this problem is to realize that F, L , and I now make up a block. You may also realize that the block cannot go with any of the others because I cannot be with H or M, K must be alone, and FLI block cannot go with G (because then U would have four, S would not have more than T, as required by the rules.) So, the FLI block cannot be on T or U. Looking at the diagram that contains that information and looking for an answer choice that violates that diagram will lead you to answer choice C.
 medialaw111516
  • Posts: 80
  • Joined: Dec 11, 2018
|
#71946
Makes sense, thanks Claire!

Get the most out of your LSAT Prep Plus subscription.

Analyze and track your performance with our Testing and Analytics Package.